Difference between revisions of "1961 IMO Problems/Problem 4"

(posted problem from IMO Compendium)
 
m (needsolution)
Line 1: Line 1:
 +
==Problem==
 
In the interior of triangle <math>ABC</math> a point ''P'' is given.  Let <math>Q_1,Q_2,Q_3</math> be the intersections of <math>PP_1, PP_2,PP_3</math> with the opposing edges of triangle <math>ABC</math>.  Prove that among the ratios <math>\frac{PP_1}{PQ_1},\frac{PP_2}{PQ_2},\frac{PP_3}{PQ_3}</math> there exists one not larger than 2 and one not smaller than 2.
 
In the interior of triangle <math>ABC</math> a point ''P'' is given.  Let <math>Q_1,Q_2,Q_3</math> be the intersections of <math>PP_1, PP_2,PP_3</math> with the opposing edges of triangle <math>ABC</math>.  Prove that among the ratios <math>\frac{PP_1}{PQ_1},\frac{PP_2}{PQ_2},\frac{PP_3}{PQ_3}</math> there exists one not larger than 2 and one not smaller than 2.
 +
==Solution==
 +
{{solution}}

Revision as of 10:36, 9 April 2007

Problem

In the interior of triangle $ABC$ a point P is given. Let $Q_1,Q_2,Q_3$ be the intersections of $PP_1, PP_2,PP_3$ with the opposing edges of triangle $ABC$. Prove that among the ratios $\frac{PP_1}{PQ_1},\frac{PP_2}{PQ_2},\frac{PP_3}{PQ_3}$ there exists one not larger than 2 and one not smaller than 2.

Solution

This problem needs a solution. If you have a solution for it, please help us out by adding it.